INBDE SPRING D2

अब Quizwiz के साथ अपने होमवर्क और परीक्षाओं को एस करें!

D) Answers A and B are both correct

6 year male patient comes into the office for a scheduled cleaning. Parent states child is very scared and acts out when he goes to his pediatrician. The best way to deal with this patient is: A) Build trust and rapport B) Minimize fear and anxiety using "Tell Show Do" C) Reschedule the appointment D) Answers A and B are both correct E) None of the above

A) Staphylococcus aureus may be the infective agent D) The tricuspid valve is more affected in these cases than the mitral valve EXPLANATION: Staphylococcus aureus may be the infective agent. Correct. Staphylococcus aureus is most often the infective agent in infective endocarditis in I.V. drug abusers, although other microbial or fungal agents may be also involved. The tricuspid valve is more affected in these cases than the mitral valve. Correct. The right side of the heart is more affected than the left side due to the direction of blood flow coming to the heart from the systemic circulation. Acute bacterial endocarditis could be suspected only if the patient had a previous rheumatic valvular lesion. Incorrect. Acute bacterial endocarditis may occur on previous healthy valves. I.V. drug abusers develop only prosthetic valve endocarditis. Incorrect. The conjunctival hemorrhages represent the major criteria for establishing the diagnosis of infective endocarditis. Incorrect. The presence of conjunctival hemorrhages represents a minor criteria for the diagnosis of IE.

A 30-year-old patient has been known as an I.V. drug abuser. He presents this time to the emergency room with malaise, fever, chills, conjunctival hemorrhages, nail-bed hemorrhages and hematuria. Which of the following is / are most likely consistent with this history and clinical presentation? Select all correct answers. A) Staphylococcus aureus may be the infective agent B) Acute bacterial endocarditis could be suspected only if the patient had a previous rheumatic valvular lesion C) I.V. drug abusers develop only prosthetic valve endocarditis D) The tricuspid valve is more affected in these cases than the mitral valve E) The conjunctival hemorrhages represent the major criteria for establishing the diagnosis of infective endocarditis

E) Stenosis of pulmonic valve EXPLANNATION: Stenosis of pulmonic valve. Correct. The narrowing of the pulmonic valve explains the systolic murmur. The stenosis at this level also explains the increased pressure in the right ventricle and, in time, the enlargement of the right ventricle because it cannot properly empty during systole. Saddle pulmonary embolism. Incorrect. The history and clinical presentation are not consistent with this severe condition. Anaphylaxis. Incorrect. The history and clinical presentation are not consistent with this severe condition. Aortic stenosis. Incorrect. It is associated with left ventricular hypertrophy. Mitral stenosis. Incorrect. It would present with a diastolic murmur and enlarged left atrium.

A 38-year-old female patient presents to the medical office for a mild allergic reaction, most likely a contact dermatitis. She has no other major complaints and does not appear to be in distress. During the physical examination, a systolic murmur is detected. Further investigation demonstrates some enlargement of the right ventricle. Which of the following is most likely the diagnosis? A) Anaphylaxis B) Saddle pulmonary embolism C) Mitral stenosis D) Aortic stenosis E) Stenosis of pulmonic valve

A) Lesser petrosal EXPLANATION: Lesser petrosal is correct. The lesser petrosal nerve forms from fibers of the tympanic branch of the glassopharyngeal in the tympanic plexus of the middle ear. It carries parasympathetic secretomotor fibers which will synapse at the otic ganglion and travel to the parotid gland, and is responsible for saliva secretion in that gland. Facial is incorrect. The facial nerve has parasypathetic secretomotor fibers to the submandibular andsublingual glands. It is not, however, secretomotor to the parotid gland. Greater petrosal is incorrect. The greater petrosal nerve is a branch of the facial nerve. It carries parasympathetic secretomotor fibers to the pterygopalatine ganglion, and postsynaptically those fibers go to glands in the mandibular region (e.g. the submandibular and sublingual glands). It is not, however, secretomotor to the parotid gland. Auriculotemporal is incorrect. The auriculotemporal nerve travels to the parotid gland. Although parasympathetic secretomotor fibers from the lesser petrosal pathway travel with it after synapsing in the otic ganglion, the auriculotemporal nerve itself is general sensory. Chorda tympani is incorrect. The chorda tympani is a branch of the facial nerve. It carries parasympathetic secretomotor fibers to the submandibular ganglion, and postsynaptically those fibers may go to salivary glands in the maxillary region. It is not, however, secretomotor to the parotid gland.

A 42-year-old female presents with a complaint of bilateral pain in the temporomandibular joint on closing. She reports that her ears are occasionally "clogged", and she sometimes experiences a "ringing" sensation. The dentist determines that these symptoms are related to her TMJ condition. Examination reveals diminished parotid salivation due to middle ear involvement. Which of the following nerves is MOST likely involved? A) Lesser petrosal B) Auriculotemporal C) Chorda tympani D) Greater petrosal

A) Approximately 6 days EXPLANTION: (google) The time to reach steady state is defined by the elimination half-life of the drug. After 1 half-life, you will have reached 50% of steady state. After 2 half-lives, you will have reached 75% of steady state, and after 3 half-lives you will have reached 87.5% of steady state. The rule of thumb is that steady state will be achieved after 5 half-lives (97% of steady state achieved).

A 45-year-old man is diagnosed as being in congestive heart failure. It is noted, however, that his renal function is relatively normal. If treatment began with oral digoxin (half-life = 36 hours) with a usual daily dosage of 0.125 mg, how long does it take for the drug to reach the steady-state concentration? A) Approximately 6 days B) Approximately 3 days C) Approximately 4 days D) Approximately 12 days

B) Ventricular septal defect EXPLANATION: Ventricular septal defect is correct. The characteristics of the murmur and thrill, as well as the general history, support the diagnosis of ventricular septal defect. Transposition of the great vessels is incorrect. Transposition of the great vessels is easy to exclude as it manifests with cyanosis. Persistent truncus arteriosus is incorrect. Persistent truncus arteriosus is easy to exclude as it manifests with cyanosis. Severe Tetralogy of Fallot is incorrect. TOF includes VSD but is more complex than an isolated VSD. In more severe cases the newborn / infant is cyanotic all the time or at least often, especially during crying, feeding or defecating. Failure to gain weight and dyspnea on exertion are also common and do not corroborate with the history of this infant. Patent foramen ovale is incorrect. The infant would have no murmur with patent foramen ovale.

A one-month-old infant appears healthy and has a history of normal development since birth. The pregnancy / delivery history was also uneventful in this case. The physical examination reveals normal first and second cardiac sounds and a loud systolic (holosystolic) murmur on the left side of the sternum, accompanied by a systolic thrill. The most likely diagnosis is: A) Severe Tetralogy of Fallot B) Ventricular septal defect C) Persistent truncus arteriosus D) Patent foramen ovale E) Transposition of the great vessels

B) Azithromycin EXPLANATION: Azithromycin is correct. Azithromycin binds to the 23S rRNA of the bacterial 50S ribosomal subunit. It stops bacterial protein synthesis. It can serve as a preferred alternative for the patient. Amoxicillin is incorrect. Like penicillin, amoxicillin belongs to the beta-lactam family. Patients that are hypersensitive to penicillin are very likely to be hypersensitive to amoxicillin as well. Tetracycline is incorrect. Tetracycline is avoided due to the discoloration of teeth. Cephalosporin is incorrect. Like penicillin, cephalosporin belongs to the beta-lactam family. Patients that are hypersensitive to penicillin are very likely to be hypersensitive to cephalosporin as well. Nystatin is ncorrect. Nystatin is an antifungal medication, irrelevant and ineffective in treating bacterial infection.

A patient has a periapical abscess and a history of hypersensitivity to penicillin. Which of the following antimicrobial agents might be the preferred alternative for this patient? A) Amoxicillin B) Azithromycin C) Tetracycline D) Cephalosporin E) Nystatin

D) large quantities of urine that is hypotonic EXPLANATION: Large quantities of urine that is hypotonic. Correct. This is diabetes insipidus, and the problem is with water reabsorption in the collecting ducts. A lower-than-normal volume of urine. Incorrect. This is the opposite of diabetes insipidus. Large quantities of urine that is hypertonic because it contains a high glucose concentration. Incorrect. This is the case in diabetes mellitus, which has no relation to ADH. Large quantities of urine that is hypertonic because it contains an above-normal Na+ concentration. Incorrect. Low or defective ADH leads to a large quantity of dilute urine. Normal quantities of urine that contains glucose. Incorrect. There is no connection between ADH and glucose

A person whose gene for antidiuretic hormone is defective will excrete: A) large quantities of urine that is hypertonic because it contains a high glucose concentration. B) large quantities of urine that is hypertonic because it contains an above-normal Na+ concentration. C) normal quantities of urine that contains glucose. D) large quantities of urine that is hypotonic. E) a lower-than-normal volume of urine.

A) Granular radiopaque and D) Well defined EXPLANATION: Granular radiopaque is correct. The pattern inside the entity shows greater and shorter trabeculation. Well-defined is correct. Considering that we can draw a line around the borders of this entity this is a well-defined lesion. Unilocular radiolucent is incorrect. The entity is not radiolucent. Multilocular radiolucent is incorrect. The entity is not radiolucent. Ill-defined is incorrect. Considering that we can draw a line around the borders of this entity this is not an ill-defined lesion

Background/or patient history: Patient doesn't have any pain or discomfort. Patient reports recent endodontic treatment on tooth # 30. Current Findings: Selected periapical radiographs were acquired based on your clinical evaluation. Teeth # 25 and # 26 are vital What is the best radiographic terminology to describe the radiographic features of the entity noted around the apices of teeth # 25 and # 26? (Select all that apply) A) Granular radiopaque B) Unilocular radiolucent C) Multilocular radiolucent D) Well-defined E) Ill-defined

A) Periapical cemento-osseous dysplasia EXPLANATION: Periapical cemento-osseous dysplasia is correct. Considering that patient is asymptomatic and teeth # 25 and # 26 are vital, the radiographic diagnosis for the well-defined and granular radiopacity noted around the apices of these teeth is periapical cemento-osseous dysplasia. Periapical condensing osteitis is incorrect. Teeth # 25 and # 26 are vital. That is why condensing osteitis is not the diagnosis. Periapical rarefying osteitis is incorrect. Teeth # 25 and # 26 are vital. That is why condensing osteitis is not the diagnosis. Also, rarefying osteitis is radiolucent on radiographs, not radiopaque. Radicular cystis incorrect. Teeth # 25 and # 26 are vital. That is why condensing osteitis is not the diagnosis. Radicular cysts are also radiolucent not radiopaque. Mental fossa is incorrect. This is not a normal anatomy

Background/or patient history: Patient doesn't have any pain or discomfort. Patient reports recent endodontic treatment on tooth # 30. Current Findings: Selected periapical radiographs were acquired based on your clinical evaluation. Teeth # 25 and # 26 are vital What is the most likely radiographic and clinical diagnosis for the entity noted around the apices of teeth # 25 and # 26? A) Periapical cemento-osseous dysplasia B) Periapical condensing osteitis C) Radicular cyst D) Periapical rarefying osteitis E) Mental fossa

D) Lingual foramen EXPLANATION: Lingual foramen is correct. A lingual foramen is a foramen on the lingual surface of the midline of the mandible in the region of the genial tubercles. Genial tubercle is incorrect. Genial tubercles are radiopacities around the lingual foramen. Nutrient canals is incorrect. Nutrient canals carry a neurovascular bundle and appear as vertical radiolucent lines of fairly uniform width. Mental fossa is incorrect. The mental fossa would be a depression on the labial surface of the mandible above the mental ridge showing a larger radiolucency. Mental foramen is incorrect. The mental foramen is the anterior limit of the inferior dental canal that is apparent on periapical radiographs. Its image is quite variable and usually in the premolars area

Background/or patient history: Patient doesn't have any pain or discomfort. Patient reports recent endodontic treatment on tooth # 30. Current Findings: Selected periapical radiographs were acquired based on your clinical evaluation. Teeth # 25 and # 26 are vital What is the name of the round radiolucency noted by a white arrow? A) Mental foramen B) Mental fossa C) Nutrient canals D) Lingual foramen E) Genial tubercle

D) No treatment required EXPLANATION: No treatment required is correct. Considering the diagnosis of periapical cemento-osseous dysplasia, no treatment is necessary.

Background/or patient history: Patient doesn't have any pain or discomfort. Patient reports recent endodontic treatment on tooth # 30. Current Findings: Selected periapical radiographs were acquired based on your clinical evaluation. Teeth # 25 and # 26 are vital What is the treatment for the entity noted in the anterior right mandible? A) Extraction of involved teeth B) Biopsy C) Endodontic treatment D) No treatment required

D) Tooth #T EXPLANATION: Primary second molar = T

Based on the panoramic film, which of the following teeth has an existing Mesial-Occlusal amalgam restoration? A) Tooth #30 B) Tooth #L C) Tooth #19 D) Tooth #T E) Tooth #S F) Tooth #K

C) Peridens; D) Distodens EXPLANATION: Peridens is correct. Peridens is a supernumerary tooth that is present in the premolar area. A peridens is present in the mandibular right premolar area Distodens is correct. Distodens is a supernumerary tooth that is present in the molar area. A distodens is present distal to tooth #1

Based on your evaluation of the panoramic radiograph, what type of dental anomaly is present? (Select all that apply) A) Fusion B) Mesiodens C) Peridens D) Gemination D) Distodens

A) Alcohol consumption EXPLANATION: Alcohol consumption is correct. Alcohol use is a risk factor for oral squamous cell carcinoma. Human papillomavirus (HPV) infection is incorrect. HPV is a significant risk factor for oropharyngeal cancer, but is not as strongly linked to cancer of the oral cavity. Family history of oral cancer is incorrect. The vast majority of oral cancers are not linked to a heritable condition. Cannabis use is incorrect. Cannabis use has not been associated with a higher risk of oral cancer

Biopsy confirms a diagnosis of squamous cell carcinoma. Which of the following is the most significant risk factor for this disease? A) Alcohol consumption B) Cannabis use C) Human papillomavirus (HPV) infection D) Family history of oral cancer

B) Saliva EXPLANATION: Saliva is correct. Saliva does NOT transmit HIV, unless it involves visible blood, such as from bleeding gums or mouth sores. Semen is incorrect. Semen can transmit HIV. Serum is incorrect. Blood serum can transmit HIV. Breast milk is incorrect. Mother-to-child transmission can occur during pregnancy, birth, or breastfeeding.

Each of the following fluids is considered one that can transmit HIV EXCEPT one. Which one is the EXCEPTION? A) Serum B) Saliva C) Breast milk D) Semen

B) Contains minimal bacteria

Firm dentin: A) Has largely denatured collagen fibrils B) Contains minimal bacteria C) Is easily removed with hand instruments D) Has a higher moisture content than soft dentin

E) Tachycardia EXPLANATION: Tachycardia. Correct. If the LV-EDV / stroke volume is reduced, the heart will try to compensate by increasing the heart rate in order to maintain a cardiac output as close to normal as possible. Increased stroke volume. Incorrect. Decreased due to hypovolemia. Drastic changes in the levels of hemoglobin and hematocrit. Incorrect. Although these changes occur, they may not be initially detected or at least drastic. Increased left ventricle end diastolic volume. Incorrect. Decreased due to hypovolemia. Decreased peripheral vascular resistance. Incorrect. The peripheral vascular resistance is increased in this case due to arteriolar vasoconstriction

Following an accident at work, a 43-year-old patient is brought by his colleagues to the emergency room. He presents with a laceration on his left lower limb. The blood loss due this wound is estimated to be mild to moderate. Which of the following is most likely expected in this case before any treatment is instituted? A) Increased left ventricle end diastolic volume B) Decreased peripheral vascular resistance C) Increased stroke volume D) Drastic changes in the levels of hemoglobin and hematocrit E) Tachycardia

C) 5

For drug X, the TD50 is 50 mg and the ED50 is 10 mg. What is the therapeutic index? A) 60 B) 40 C) 5 D) 0.5

B) TIMPs inhibit MMP activity in the extracellular matrix. EXPLANATION: TIMPs inhibit MMP activity in the extracellular matrix. Correct. MMPs (matrix metalloproteinases) degrade proteins of the extracellular matrix. They require zinc as a co-factor. TIMP stands for "tissue inhibitor of metalloproteinases". TIMPs prevent MMPs from binding with zinc, and therefore inhibit MMP activity. TIMPs increase MMP activity in the extracellular matrix. Incorrect. Tissue inhibitors of metalloproteinases (TIMPs) will inhibit MMPs. TIMPs have no effect on MMP activity. Incorrect. Tissue inhibitors of metalloproteinases (TIMPs) will inhibit MMPs. TIMPs inhibit MMP activity in the cytoplasm. Incorrect. MMPs and TIMPs are active outside of the cell. TIMPs increase MMP activity in the cytoplasm. Incorrect. Tissue inhibitors of metalloproteinases (TIMPs) will inhibit MMPs. Also, MMPs and TIMPs are active outside of the cell.

How do the TIMP proteins (TIMPs 1-4) affect the activity of MMPs? A) TIMPs increase MMP activity in the extracellular matrix. B) TIMPs inhibit MMP activity in the extracellular matrix. C) TIMPs have no effect on MMP activity. D) TIMPs inhibit MMP activity in the cytoplasm. E) TIMPs increase MMP activity in the cytoplasm.

C) deposits of a gummy polysaccharide called glucan. EXPLANATION: Deposits of a gummy polysaccharide called glucan is correct. The bacterium synthesizes adhesive glucan from sucrose by the action of glycosyltransferase. L-forms is incorrect. L-forms indicate a lack of cell walls, while S. mutans are Gram-positive bacteria. Very long chains of cocci is incorrect. S. mutans, the major caries pathogen, commonly aggregates into pairs and short chains. Regular diploid pairs rather than chains is incorrect. S. mutans, the major caries pathogen, commonly aggregates into pairs and short chains. A very thick wall of muramic acid and mucopeptide substances is incorrect. S. mutans' cell wall is composed of peptidoglycan (murein/mucopeptide) and teichoic acids.

In the presence of sucrose, Streptococcus mutans produces A) very long chains of cocci. B) a very thick wall of muramic acid and mucopeptide substances. C) deposits of a gummy polysaccharide called glucan. D) regular diploid pairs rather than chains. E) L-forms

A) Sensoryneural deafness EXPLANATION: Sensoryneural deafness is correct. This refers to the loss of hearing due to malfunction of inner ear structures (i.e., cochlea or auditory nerve). Mondini malformation is a condition where the cochlea of the inner ear fails to make its normal spiral turns. This deformity of the cochlea leads to a hearing loss. Congenital cataract is incorrect. This refers to having a cloudy lens at birth, and it can cause blindness. Retinal degeneration is incorrect. This refers to deterioration of the retina due to cell death. It can start at birth or later in life. It is a leading cause of blindness. Conductive deafness is incorrect. This refers to the loss of hearing due to abnormalities of the external ear and/or the ossicles of the middle ear

Mondini malformation can lead to: A) Sensoryneural deafness B) Congenital cataract C) Conductive deafness D) Retinal degeneration

A) Among dentists who do not have COVID-19 at baseline, the percentage who will develop COVID-19 after a certain follow-up period EXPLANATION: B) incidence C) prevalence

Over millions got either infected with COVID-19 or died. Oral health care providers are at a higher risk of getting infected with COVID-19 due to their proximity to the oral cavities of patients, longer procedures, and aerosol generating procedures What is meant by "risk of getting infected with COVID-19"? A) Among dentists who do not have COVID-19 at baseline, the percentage who will develop COVID-19 after a certain follow-up period. B) The rate of new COVID-19 cases emerging from a dentist population over a certain period. C) The percentage of COVID-19 cases among dentists in a certain area at a certain time.

B) Proportion brush biopsy positive among those with oral cancer EXPLANATION: Sensitivity is how many are positive in your test if you apply the test to people with the disease

Oral cancer is a considerable public health problem. Early detection will help better manage these patients and enhance their survival. Among the screening tests we can use to early detect oral cancer is a brush biopsy. Compared to traditional histopathology, brush biopsy has a sensitivity of 84% of detecting oral cancer. What is meant by "sensitivity" in the above statement? A) Proportion with no oral cancer among brush biopsy negative B) Proportion brush biopsy positive among those with oral cancer C) Proportion brush biopsy negative among those without oral cancer D) Proportion with oral cancer among brush biopsy positive

D EXPLANATION: D correct: this is healthy nonkeratinized stratified squamous epithelium A is in correct: the large basophilic mass filling the right side of the image is pathological and would not be found in healthy tissue in this region B is incorrect: this is healthy simple cuboidal epithelium C is incorrect: this is healthy pseudostratified ciliated columnar epithelium E is incorrect: this is healthy compact bone (specifically, an osteon)

Patient: 45-year-old man Chief complaint: "There's this weird thing under my tongue." Background/or patient history: No significant history Which of the following images shows epithelium from the underside of the left side of the tongue in the patient image?

C) Cattle EXPLANATION: Cattle is correct. Cattle are the major source of the STEC that cause human illness. Bats is incorrect. Bats are reservoirs of different kinds of viruses. Ticks is incorrect. Ticks are vectors of pathogens including Borrelia burgdorferi and Rickettsia. Mosquitos incorrect. Mosquitoes are vectors of many pathogens, including Plasmodium (malaria), West Nile virus, Dengue virus, Zika virus, etc.

Patient: 11-year-old female Chief Complaint: "I've had bloody diarrhea from days ago. My diarrhea began as watery and became increasingly bloody. I started to have fever, headache, and lower abdominal pain." Background/Patient History: Previously healthy. There was no family history of inflammatory bowel disease or bloody stools. No recent travel. Prior to the onset of disease, she had eaten a hamburger and some spinach at a school picnic. Current Findings: Red blood cells and white blood cells were tested in her stool. An abdominal ultrasound ruled out acute appendicitis but revealed colitis. The patient later developed hematuria and kidney disease. Culture of her stool on sorbitol MacConkey agar showed colorless colonies. The organism is further tested as Gram-negative lactose-fermenter. What is the major source of this organism? A) Bats B) Ticks C) Cattle D) Mosquitos

A) Shiga toxin (Stx) EXPLANATION: Shiga toxin (Stx) is correct. Stx1 and Stx2 are the AB5 toxins. The A subunit inhibits protein synthesis, causes apoptosis, and promotes expression of pro-inflammatory cytokines. It causes damage to the intestinal epithelium. Heat-labile toxin (LT) is incorrect. LT is AB5 toxin produced by ETEC. It causes hypersecretion of water and electrolytes, and non-bloody diarrhea. Staphylococcal enterotoxin is incorrect. Secreted by S. aureus. Ingestion results in nausea, vomiting, stomach cramps and sometimes diarrhea. The symptoms develop within hours, and usually do not last long. Cholera toxin (Ctx) is incorrect. Ctx is AB5 toxin produced by V. cholerae. It is similar to LT

Patient: 11-year-old female Chief Complaint: "I've had bloody diarrhea from days ago. My diarrhea began as watery and became increasingly bloody. I started to have fever, headache, and lower abdominal pain." Background/Patient History: Previously healthy. There was no family history of inflammatory bowel disease or bloody stools. No recent travel. Prior to the onset of disease, she had eaten a hamburger and some spinach at a school picnic. Current Findings: Red blood cells and white blood cells were tested in her stool. An abdominal ultrasound ruled out acute appendicitis but revealed colitis. The patient later developed hematuria and kidney disease. Culture of her stool on sorbitol MacConkey agar showed colorless colonies. The organism is further tested as Gram-negative lactose-fermenter. What virulence factor does this organism produce that could explain her clinical symptoms? A) Shiga toxin (Stx) B) Staphylococcal enterotoxin C) Heat-labile toxin (LT) D) Cholera toxin (Ctx)

A) STEC (Shiga toxin producing E. Coli) EXPLANATION: STEC (Shiga toxin producing E. Coli) is correct. This gram-negative, lactose-fermenting rod can cause bloody diarrhea and HUS. Transmitted by consuming contaminated meat or vegetable. Helicobacter pylori is incorrect. This gram-negative, curved rod can cause peptic ulcers. Staphylococcus aureus is incorrect. This gram-positive coccus can produce toxin and cause food poisoning. Vibrio cholera is incorrect. Vibrio cholera is gram-negative curved rod, which can cause cholera with patients presenting with or dying from watery diarrhea.

Patient: 11-year-old female Chief Complaint: "I've had bloody diarrhea from days ago. My diarrhea began as watery and became increasingly bloody. I started to have fever, headache, and lower abdominal pain." Background/Patient History: Previously healthy. There was no family history of inflammatory bowel disease or bloody stools. No recent travel. Prior to the onset of disease, she had eaten a hamburger and some spinach at a school picnic. Current Findings: Red blood cells and white blood cells were tested in her stool. An abdominal ultrasound ruled out acute appendicitis but revealed colitis. The patient later developed hematuria and kidney disease. Culture of her stool on sorbitol MacConkey agar showed colorless colonies. The organism is further tested as Gram-negative lactose-fermenter. Which of the following bacteria is the most likely cause of the infection? A) STEC (Shiga toxin producing E. Coli) B) Staphylococcus aureus C) Vibrio cholera D) Helicobacter pylori

C) O157:H7 EXPLANATION: O157:H7 is correct. O157:H7 is sorbitol-negative. This serotype of E. coli is associated with bloody diarrhea. O157:H7 appears to be the most common cause of this illness in the Northern Hemisphere. H1N1 is incorrect. A subtype of influenza virus. O1 is incorrect. Serotype O1 of V. cholerae is mainly responsible for cholera pandemics

Patient: 11-year-old female Chief Complaint: "I've had bloody diarrhea from days ago. My diarrhea began as watery and became increasingly bloody. I started to have fever, headache, and lower abdominal pain." Background/Patient History: Previously healthy. There was no family history of inflammatory bowel disease or bloody stools. No recent travel. Prior to the onset of disease, she had eaten a hamburger and some spinach at a school picnic. Current Findings: Red blood cells and white blood cells were tested in her stool. An abdominal ultrasound ruled out acute appendicitis but revealed colitis. The patient later developed hematuria and kidney disease. Culture of her stool on sorbitol MacConkey agar showed colorless colonies. The organism is further tested as Gram-negative lactose-fermenter. Which serotype of this organism is the most likely cause of the infection? A) O1 B) H1N1 C) O157:H7

A) Hyperglycemia C) Dehydration EXPLANATION: Hyperglycemia is correct. Long-term hyperglycemia can result in the formation of reactive oxygen species (ROS) and advanced glycation end products (AGE) which have several negative impacts on cardiovascular health. Dehydration is correct. The patient is likely severely dehydrated since increased lipolysis and gluconeogenesis results in osmotically active glucose and ketone bodies in the urine. Glycosuria is incorrect. Elevated levels of blood glucose ultimately cause excess glucose to be excreted in the urine. However, this symptom in itself is not a significant complication. Cataracts is incorrect. While this is a potential long-term complication resulting from diabetes, the patient is likely too young to be impacted by cataracts. Atherosclerosis is incorrect. While this is a potential long-term complication resulting from diabetes, the patient is likely too young to be impacted by arterial complications at this point.

Patient: 12 year-old-female Chief complaint of parent : Help! My child is unconscious! Background/or patient history: The patient was recently diagnosed with Type I diabetes and has since been on a regimen of exogenous insulin. Current Findings: The patient is suspected of having missed an insulin dose following a meal. She was reported by her mother to be acting confused and had a distinct strong odor coming from her body. The parent rushed her child to the emergency room, where she presented as unconscious. The patient is diagnosed as suffering from a severe case of ketoacidosis Along with ketoacidosis, what other medical conditions are also likely significantly impacting the patient? Select all that apply A) Hyperglycemia B) Glycosuria C) Dehydration D) Atherosclerosis E) Cataracts

C) Acetyl-CoA EXPLANATION: Acetyl-CoA is correct. A buildup of acetyl-CoA results in the formation of ketone bodies which ultimately leads to ketoacidosis if left unchecked. Glucose is incorrect. While blood glucose levels may be elevated in a patient with Type 1 diabetes, the resulting symptoms would not be due to hyperglycemia. Pyruvate is incorrect. Any pyruvate in the patient's system would likely be going towards synthesizing glucose through the gluconeogenic pathway. Oxaloacetate is incorrect. Any oxaloacetate in the patient's system would likely be going towards synthesizing glucose through the gluconeogenic pathway. Palmitic acid is incorrect. While palmitic acid is eventually broken down through beta-oxidation to acetyl-CoA, the mobilization of fatty acids from adipocytes would not cause the observed symptoms.

Patient: 12 year-old-female Chief complaint of parent : Help! My child is unconscious! Background/or patient history: The patient was recently diagnosed with Type I diabetes and has since been on a regimen of exogenous insulin. Current Findings: The patient is suspected of having missed an insulin dose following a meal. She was reported by her mother to be acting confused and had a distinct strong odor coming from her body. The parent rushed her child to the emergency room, where she presented as unconscious. The patient is diagnosed as suffering from a severe case of ketoacidosis Which of the following metabolites present in excess is most directly responsible for the observed symptoms? A) Pyruvate B) Oxaloacetate C) Acetyl-CoA D) Glucose E) Palmitic acid

A EXPLANATION: A is correct: the large basophilic mass filling the right side of the image is pathological and would not be found in healthy tissue in this region B is incorrect: this is healthy simple cuboidal epithelium C is incorrect: this is healthy pseudostratified ciliated columnar epithelium D is incorrect: this is healthy nonkeratinized stratified squamous epithelium E is incorrect: this is healthy compact bone (specifically, an osteon)

Patient: 45-year-old man Chief complaint: "There's this weird thing under my tongue." Background/or patient history: No significant history. Which of the following images shows a biopsy of the structure on the underside of the right side of the tongue in the patient image?

A) Glucagon C) Epinephrine EXPLANATION: Epinephrine is correct. Muscle tissue is stimulated by epinephrine during a low-insulin metabolic state rather than glucagon. Glucagon is correct. In the liver, glucagon is signaling to hepatocyte to breakdown fatty acids to acetyl-CoA and to utilize TCA cycle intermediates for gluconeogenesis. Endogenous insulin is incorrect. As the patient is a Type I diabetic, they are unable to produce their own insulin. Cortisol is incorrect. This hormone plays a role in glucose metabolism and managing the stress response, but it not as important as other regulatory hormones. Dopamine is incorrect. This neurotransmitter governs many vital functions in the body but is not directly responsible for the patient's symptoms.

Patient: 12 year-old-female Chief complaint of parent : Help! My child is unconscious! Background/or patient history: The patient was recently diagnosed with Type I diabetes and has since been on a regimen of exogenous insulin. Current Findings: The patient is suspected of having missed an insulin dose following a meal. She was reported by her mother to be acting confused and had a distinct strong odor coming from her body. The parent rushed her child to the emergency room, where she presented as unconscious. The patient is diagnosed as suffering from a severe case of ketoacidosis. Other than exogenous insulin, which hormone(s) is primarily driving the metabolic state of the patient? Select all that apply A) Glucagon B) Cortisol C) Epinephrine D) Endogenous insulin E) Dopamine

A) Beta-hydroxybutyrate B) Acetoacetate EXPLANATION: Acetoacetate is correct. Excess acetoacetate readily diffuses from the liver into the blood where its buildup causes an imbalance in the blood-buffering system and ultimately a significant decrease in blood pH. Beta-hydroxybutyrate is correct. Excess beta-hydroxybutyrate readily diffuses from the liver into the blood where its buildup causes an imbalance in the blood-buffering system and ultimately a significant decrease in blood pH. Acetoacetyl-CoA is incorrect. This metabolite is an intermediate formed during ketogenesis. Acetone is incorrect. While acetone is an end product of ketogenesis, it does not reduce the pH of the blood significantly compared to other ketone bodies. Oxaloacetate is incorrect. While technically a ketoacid, oxaloacetate is a key metabolite in both the TCA cycle and gluconeogenesis so it would not build up significantly.

Patient: 12 year-old-female Chief complaint of parent : Help! My child is unconscious! Background/or patient history: The patient was recently diagnosed with Type I diabetes and has since been on a regimen of exogenous insulin. Current Findings: The patient is suspected of having missed an insulin dose following a meal. She was reported by her mother to be acting confused and had a distinct strong odor coming from her body. The parent rushed her child to the emergency room, where she presented as unconscious. The patient is diagnosed as suffering from a severe case of ketoacidosis. Which of the following molecules in the patient's bloodstream are causing her life-threatening condition? Select all that apply. A) Beta-hydroxybutyrate B) Acetoacetate C) Acetone D) Acetoacetyl-CoA E) Oxaloacetate

B) Megaloblastic anemia EXPLANATION: Megaloblastic anemia is correct. Megaloblastic anemia or macrocytic anemia is characterized by larger than normal and fewer red blood cells. Vitamin B12 deficiency and folate deficiency are the common cause of megaloblastic anemia. Megaloblastic anemias can be diagnosed based on morphologic and laboratory findings. A mean corpuscular value (MCV) > 100 indicates macrocytosis. The bone marrow produces large immature red cells, and DNA synthesis is impaired due to vitamin B12 or folate deficiency. Aplastic anemia is incorrect. Aplastic anemia develops as a result of bone marrow damage from different etiologies. In aplastic anemia all blood cells are lower than the normal range. Microcytic anemia is incorrect. Microcytic anemia is defined with the presence of smaller red blood cells than normal, usually hypochromic. Iron deficiency is the most common cause of microcytic hypochromic anemia. In microcytic anemias the mean corpuscular value (MCV) is < 80 femtolitres (fL). Normocytic anemia is incorrect. In normocytic anemia, the red blood cells have a normal size but a lower number than normal. Anemia of chronic inflammation/disease is the most common normocytic anemia. The normal mean corpuscular value (MCV) is between 80-100 femtolitres (fL)

Patient: 25-year old female Chief complaint: "My mouth hurts in particular when I'm eating certain foods. I feel a burning sensation." Background/Patient history: Heartburn for which she takes daily over the counter (OTC) medication: omeprazole. Strict vegetarian for the last 8 years. Current findings: Pale oral mucosa, glossitis with papillary atrophy and multiple areas of painful erythema on the dorsal surface and lateral borders of the tongue. A hematological test was requested to assist on the suspected diagnosis Based on the etiology of her condition and the current laboratory values, what is the type of anemia that this patient is presenting? A) Microcytic anemia B) Megaloblastic anemia C) Aplastic anemia D) Normocytic anemia

A) Vitamin B12 deficiency

Patient: 25-year old female Chief complaint: "My mouth hurts in particular when I'm eating certain foods. I feel a burning sensation." Background/Patient history: Heartburn for which she takes daily over the counter (OTC) medication: omeprazole. Strict vegetarian for the last 8 years. Current findings: Pale oral mucosa, glossitis with papillary atrophy and multiple areas of painful erythema on the dorsal surface and lateral borders of the tongue. A hematological test was requested to assist on the suspected diagnosis Based on the laboratory values, what do you think is the etiology of the patient's current condition? A) Vitamin B12 deficiency B) Iron deficiency C) Folate deficiency

F) all of the above EXPLANATION: Neurological complications is correct. Vitamin B12 is important for neural function. Prolonged and severe vitamin deficiency can cause neurological and hematological disorders. Cognitive decline is correct. In patients with cognitive impairment, vitamin B12 should be excluded as a cause, in particular for elderly patients with signs of dementia. Paresthesia or polyneuropathy is correct. Paresthesia or polyneuropathy could be an early and subtle sign of vitamin B12 deficiency. Unsteady gait is correct. Severe and chronic vitamin B12 deficiency can cause posterior column degeneration and eventually lead to ataxic gait. Burning sensation and glossitis is correct. Oral symptoms can be present in patients with vitamin B12. Tongue pain, erythema and depapillation of the tongue are common findings.

Patient: 25-year old female Chief complaint: "My mouth hurts in particular when I'm eating certain foods. I feel a burning sensation." Background/Patient history: Heartburn for which she takes daily over the counter (OTC) medication: omeprazole. Strict vegetarian for the last 8 years. Current findings: Pale oral mucosa, glossitis with papillary atrophy and multiple areas of painful erythema on the dorsal surface and lateral borders of the tongue. A hematological test was requested to assist on the suspected diagnosis Please select all non-hematological complications that could be associated with severe vitamin B12 deficiency. Select all correct answers. A) Paresthesia or polyneuropathy B) Unsteady gait C) Cognitive decline D) Neurological complications E) Burning sensation and glossitis F) all of the above

B) Myocardial infarction EXPLANATION: Myocardial infarction is correct. Patients suffering from metabolic syndrome are at high risk of heart disease and plaque formation due to several compounding factors. In particular, increased inflammation due to the higher levels of advanced glycation end-products (AGE) makes plaque rupture and a subsequence stroke or heart attack more likely. Cataracts is incorrect. While the patient is at risk for developing diabetic cataracts, this is a potential long-term complication, and based on his medical history there is a more pressing concern that the patient must monitor closely. Neuropathy is incorrect. While the patient is at risk for diabetic neuropathy (nerve damage), this is a potential long-term complication, and based on his medical history there is a more pressing concern that the patient must monitor closely. Ketoacidosis is incorrect. While a common complication in type I diabetes, ketoacidosis typically does not occur in those with type II diabetes as they still express some endogenous insulin, but not enough to sufficiently regulate their blood glucose levels. Nephropathy is incorrect. While the patient is at risk for diabetic nephropathy (kidney disease), this is a potential long-term complication, and based on his medical history there is a more pressing concern that the patient must monitor closely.

Patient: 56-year-old male Chief complaint: I have pain in my chest when I climb stairs. Background/or patient history: The patient is obese and presented with arterial hypertension, hypertriglyceridemia, and impaired fasting glucose. The patient takes a combination of aspirin and statins to help control his high blood pressure and cholesterol. The man has a poor diet, does not exercise regularly, and has a family history of high blood pressure and other cardiovascular illnesses. Current Findings: The impaired fasting glucose suggests that the patient has developed peripheral insulin resistance, possibly type II diabetes. Administering a glucose tolerance test confirmed this diagnosis. In combination with his other symptoms, the patient is suffering from a condition known as metabolic syndrome. Given the patient's condition and medical history, which of the following complications is the patient most at risk for? A) Neuropathy B) Myocardial infarction C) Cataracts D) Ketoacidosis E) Nephropathy

C) Sorbitol EXPLANATION: Sorbitol is correct. Glucose is converted to sorbitol through aldose reductase through the polyol pathway. Aldolase reductase has a high Km for glucose and therefore is not very active at normal glucose concentrations. Sorbitol is osmotically active and its accumulation in ocular tissue contributes to the development of cataracts. Glucose is incorrect. Uncontrolled hyperglycemia can lead to diabetic cataracts, among other symptoms. However, glucose is not directly responsible for this complication. Fructose is incorrect. The pathway responsible for diabetic cataracts, the polyol pathway, produces fructose from glucose in a two-step mechanism. However, as this pathway is highly active in diabetics, a build-up of an intermediate in the pathway occurs that results in a variety of complications. Acetyl-CoA is incorrect. Insulin resistance associated with type II diabetes can result in the mobilization of fatty acids and ultimately acetyl-CoA. However, this is not directly associated with the formation of diabetic cataracts. Mannitol is incorrect. Mannitol is a sugar alcohol used as either a sweetener or a medication. As a sweetener, it is used by people with diabetes as it is poorly absorbed in the intestines. As a medication, it is used to decrease eye pressure in the case of glaucoma or to lower intracranial pressure.

Patient: 56-year-old male Chief complaint: I have pain in my chest when I climb stairs. Background/or patient history: The patient is obese and presented with arterial hypertension, hypertriglyceridemia, and impaired fasting glucose. The patient takes a combination of aspirin and statins to help control his high blood pressure and cholesterol. The man has a poor diet, does not exercise regularly, and has a family history of high blood pressure and other cardiovascular illnesses. Current Findings: The impaired fasting glucose suggests that the patient has developed peripheral insulin resistance, possibly type II diabetes. Administering a glucose tolerance test confirmed this diagnosis. In combination with his other symptoms, the patient is suffering from a condition known as metabolic syndrome. Long term, the patient's hyperglycemia may result in the formation of diabetic cataracts. Which of the following molecules would directly be responsible for this condition? A) Glucose B) Mannitol C) Sorbitol D) Acetyl-CoA E) Fructose

B) The non-enzymatic glycation of proteins EXPLANATION: The non-enzymatic glycation of proteins. Correct. Non-enzymatic, concentration-dependent glycation reactions between glucose and protein lead to the formation of glucose-protein adducts. This modification affects the structure and function of the protein. Glycated proteins are the substrates for the formation of AGE. AGE accumulation is part of the aging process, but hyperglycemia accelerates in the presence. The enzymatic glycation of proteins. Incorrect. Glycosylation is the process by which sugar moieties are added to proteins enzymatically as a post-translation modification, commonly found in proteins in the extracellular matrix. However, the glycation of proteins is the process by which proteins form concentration-dependent protein-glucose adducts. The spontaneous oxidation of glucose. Incorrect. The spontaneous oxidation of glucose does not occur under physiological conditions. The enzymatic oxidation of glucose. Incorrect. The enzymatic oxidation of glucose is what occurs naturally through glycolysis. The aggregation of glycated proteins. Incorrect. While this may be a downstream effect of protein glycation, it is not the initial step in the formation of AGE.

Patient: 56-year-old male Chief complaint: I have pain in my chest when I climb stairs. Background/or patient history: The patient is obese and presented with arterial hypertension, hypertriglyceridemia, and impaired fasting glucose. The patient takes a combination of aspirin and statins to help control his high blood pressure and cholesterol. The man has a poor diet, does not exercise regularly, and has a family history of high blood pressure and other cardiovascular illnesses. Current Findings: The impaired fasting glucose suggests that the patient has developed peripheral insulin resistance, possibly type II diabetes. Administering a glucose tolerance test confirmed this diagnosis. In combination with his other symptoms, the patient is suffering from a condition known as metabolic syndrome. The atherogenesis underpinning the patient's conditions is complicated by the formation of advanced glycation end-products (AGE). What is the first step in the process? A) The spontaneous oxidation of glucose B) The non-enzymatic glycation of proteins C) The enzymatic glycation of proteins D) The enzymatic oxidation of glucose E) The aggregation of glycated proteins

E) Hypoglycemia EXPLANATION: Hypoglycemia is correct. In adjusting to a lifestyle managing his type II diabetes, the patient may take too much insulin, miss a meal, or not properly adjust his insulin dose before strenuous activity. Hypoglycemia, characterized by blood glucose levels below 4 mmol/L, can result in sweating, tremors, tachycardia, feelings of hunger, and in extreme cases, unconsciousness and death. Hyperglycemia is incorrect. After administering insulin, it would be unlikely for the patient to be hyperglycemic unless he has administered too little insulin. Ketoacidosis is incorrect. While a common complication in type I diabetes, ketoacidosis typically does not occur in those with type II diabetes as they still express some endogenous insulin, but not enough to sufficiently regulate their blood glucose levels. Diabetic cataracts is incorrect. While this is a potential long-term complication of diabetes, it would not be associated with the patient adjusting to regulating his insulin levels. Ketonuria is incorrect. The patient may or may not have ketones in his urine due to the build-up of acetyl-CoA in his system; however, administering insulin would help to combat this symptom so it would not be likely.

Patient: 56-year-old male Chief complaint: I have pain in my chest when I climb stairs. Background/or patient history: The patient is obese and presented with arterial hypertension, hypertriglyceridemia, and impaired fasting glucose. The patient takes a combination of aspirin and statins to help control his high blood pressure and cholesterol. The man has a poor diet, does not exercise regularly, and has a family history of high blood pressure and other cardiovascular illnesses. Current Findings: The impaired fasting glucose suggests that the patient has developed peripheral insulin resistance, possibly type II diabetes. Administering a glucose tolerance test confirmed this diagnosis. In combination with his other symptoms, the patient is suffering from a condition known as metabolic syndrome. The patient will need to begin taking exogenous insulin to treat his type II diabetes. Which of the following is the most likely complication that could result from self-administering insulin? A) Ketoacidosis B) Hyperglycemia C) Diabetic cataracts D) Ketonuria E) Hypoglycemia

A) Diet and lifestyle choices B) Use of statins D) Genetic predisposition EXPLANATION: Diet and lifestyle choices. Correct. The Diabetes Prevention Program study showed a 58% decrease in the development of type II diabetes after lifestyle changes involving diet and exercise. Use of statins. Correct. A working model proposed by Mitchel and Marette describes how the prolonged used of statins to lower cholesterol increases peripheral insulin resistance. Thus, patients affected by metabolic syndrome must take additional precautions in managing their levels of blood glucose and LDL. Genetic predisposition. Correct. Though not completely understood, genetic predisposition is a likely factor that contributes to the development of type II diabetes. First-degree relatives of persons with type II diabetes have a 40% chance of developing the disease. Population testing has identified six genes and 80 loci associated with the diseases, including those associated with insulin sensitivity and secretion. Use of aspirin. Incorrect. Aspirin is commonly used in combination with statins to help reduce the risk of a cardiovascular event. Use of biguanides. Incorrect. Metformin, a biguanide, is the most common drug used to treat type II diabetes. It reduces hepatic gluconeogenesis, inhibits glycogenolysis, increases fatty acid oxidation, and increases peripheral insulin sensitivity. Metformin also increases insulin-dependent glucose updates in skeletal muscle

Patient: 56-year-old male Chief complaint: I have pain in my chest when I climb stairs. Background/or patient history: The patient is obese and presented with arterial hypertension, hypertriglyceridemia, and impaired fasting glucose. The patient takes a combination of aspirin and statins to help control his high blood pressure and cholesterol. The man has a poor diet, does not exercise regularly, and has a family history of high blood pressure and other cardiovascular illnesses.Current Findings: The impaired fasting glucose suggests that the patient has developed peripheral insulin resistance, possibly type II diabetes. Administering a glucose tolerance test confirmed this diagnosis. In combination with his other symptoms, the patient is suffering from a condition known as metabolic syndrome. Which of the following are possible reasons for the development of type II diabetes in the patient? Select all that apply. A) Diet and lifestyle choices B) Use of statins C) Use of aspirin D) Genetic predisposition E) Use of biguanides

A) To prevent embolic events --> Anticoagulation reduces the risk of ischemic stroke and other embolic events B) To prevent strokes --> Atrial fibrillation increases the risk of thromboembolic events

Patient: 78- year- old male Chief complaint: "I need a checkup. I was told that I have cavities and a tooth needs to be extracted." Background/or patient history: Hypertension and atrial fibrillation (AF). Medications: Metoprolol, amiodarone and warfarin. Current Findings: Generalized gingivitis. He needs restorative treatment on #14 and #15. #2 is not restorable. BP: 120/82 Why does this patient need to receive anticoagulation medications? (Select all correct answers.) A) To prevent embolic events B) To prevent strokes C) To increase bleeding

C) Increased renal bicarbonate production EXPLANATION: Increased renal bicarbonate production is correct. This is the correct answer since increased renal bicarbonate production is not directly linked to the activity of the respiratory system. Hypoxia is incorrect. Asthma results in the decreased rates of ventilation and as a result in decreased oxygen delivery causing hypoxia. Tachycardia is incorrect. Tachycardia is an increased heart rate. Due to the increased airway resistance the body attempts to compensate for this by the increased work of respiration which results in the increased heart rate. Hypercapnia is incorrect. Hypercapnia is an increased level of the carbon dioxide in the system. This is the direct consequence of the decreased rates of ventilation caused by the asthma attack. Due to the decrease in ventilation metabolic carbon dioxide is not completely removed from the system in timely manner resulting in hypercapnia. Acute respiratory acidosis is incorrect. Acute respiratory acidosis (decreased pH) is caused by the increased levels of the carbon dioxide (see explanation above) since the level of carbon dioxide is directly linked to the pH.

Patient: 7-year-old male Chief Complaint: Routine checkup Background / Patient History: History of asthma Current Findings: During the initial exam, the patient demonstrates acute dyspnea. The immediate consequences of this patient's respiratory difficulty include each of the following EXCEPT one. Which one is this EXCEPTION? A) Tachycardia B) Hypoxia C) Increased renal bicarbonate production D) Hypercapnia E) Acute respiratory acidosis

B) Reduce the bronchospasm in order to improve ventilation EXPLANATION: Reduce the bronchospasm in order to improve ventilation is correct. Reducing bronchospasm will help to open airways and increase the ventilation. Reduce the inflammation is incorrect. While this is an important part of treatment, reducing inflammation can not be achieved instantly, as is needed to treat the acute phase. Antibiotic treatment is incorrect. Asthma is not associated with bacterial infection and this treatment will not help.

Patient: 7-year-old male Chief Complaint: Routine checkup Background / Patient History: History of asthma Current Findings: During the initial exam, the patient demonstrates acute dyspnea. What strategy should be used immediately in order to treat this patient? A) Antibiotic treatment B) Reduce the bronchospasm in order to improve ventilation C) Reduce the inflammation

D) Decreased surfactant EXPLANATION: Decreased surfactant is correct. This is the correct choice. Surfactant is in the lining of the alveolar walls and asthma does not have direct effect on the lung surfactant. Airway edema is incorrect. Edema is the direct consequence of the asthma attack. Bronchospasm is incorrect. During an asthma attack the primary event is the contraction of the airways. Increased mucous secretion is incorrect. Mucus secretion of the body is a response to the asthma attack. Increased airway resistance is incorrect. Bronchospasm causes the contraction of the airways and as a result they decrease in their total diameter and there is a dramatic increase of the airway resistance.

Patient: 7-year-old male Chief Complaint: Routine checkup Background / Patient History: History of asthma Current Findings: During the initial exam, the patient demonstrates acute dyspnea. You would commonly expect to observe each of the following in this patient EXCEPT one. Which one is this EXCEPTION? A) Increased airway resistance B) Increased mucous secretion C) Bronchospasm D) Decreased surfactant E) Airway edema

ALL: A) Risk of medication(s) interaction with epinephrine --> amiodarone (prolong QT interval) B) Risk of bleeding --> warfarin = oral anticoagulant C) Risk of medication side effects --> amiodarone = taste distortion and abnormal salivation (xerostomia is rare) D) Risk of medication interaction with certain analgesics --> NSAIDS reduce hypotensive effects of beta-blockers (metoprolol)

Patient: 78 year-old male Chief complaint: "I need a checkup. I was told that I have cavities and a tooth needs to be extracted." Background/or patient history: Hypertension and atrial fibrillation (AF). Medications: Metoprolol, amiodarone and warfarin. Current Findings: Generalized gingivitis. He needs restorative treatment on #14 and #15. #2 is not restorable. BP: 120/82 Which of the following risks should be assessed in this patient before performing the dental extraction? Select all the correct answers A) Risk of medication(s) interaction with epinephrine B) Risk of bleeding C) Risk of medication side effects D) Risk of medication interaction with certain analgesics

A) Factor X B) Factor IX *Christmas factor* C) Factor VII D) Factor II (prothrombin)

Patient: 78 year-old male Chief complaint: "I need a checkup. I was told that I have cavities and a tooth needs to be extracted." Background/or patient history: Hypertension and atrial fibrillation (AF). Medications: Metoprolol, amiodarone and warfarin. Current Findings: Generalized gingivitis. He needs restorative treatment on #14 and #15. #2 is not restorable. BP: 120/82 Which coagulation factors are vitamin K dependent? (Select all correct answers.) A) Factor X B) Factor IX C) Factor VII D) Factor II (prothrombin)

B) International Normalized Ratio (INR) EXPLANATION: CBC is incorrect. A complete blood count is not needed to assess the risk for bleeding. In this case the medication affects the coagulation process. Warfarin is a vitamin K antagonist hence the vitamin K dependent coagulation factors are affected with use of warfarin (II,VII, IX and X) Bleeding time is incorrect. Yes, the bleeding time is increased by the medication. However, presently is not a clinically relevant test available to request prior a dental treatment Platelet count is incorrect. The number of platelets will not be affected by warfarin or any of the medications that the patient is currently taking PTT is incorrect. PTT (Partial Thromboplastin Time) is a test that was designed to measure the intrinsic pathway and the PT (Prothrombin Time) test was designed to measure the extrinsic pathway. Factor VII from the extrinsic pathway is Vitamin K dependent. PT is used because it is a more sensitive test. Warfarin typically prolongs the PT alone, but at high levels warfarin can prolong both tests

Patient: 78 year-old male Chief complaint: "I need a checkup. I was told that I have cavities and a tooth needs to be extracted." Background/or patient history: Hypertension and atrial fibrillation (AF). Medications: Metoprolol, amiodarone and warfarin. Current Findings: Generalized gingivitis. He needs restorative treatment on #14 and #15. #2 is not restorable. BP: 120/82 Which laboratory test is needed to assess the patient's bleeding risk before performing the tooth extraction? A) Platelet count B) International Normalized Ratio (INR) C) CBC D) PTT E Bleeding time

E) Amelogenesis Imperfecta, Hypoplastic type EXPLANATION: Amelogenesis Imperfecta, Hypoplastic type is correct. The presence of undersized and square shape crowns, flattening of the occlusal surfaces of the posterior teeth on the radiograph and the clinical presentation of thin enamel with a yellowish brown color on teeth are diagnostic criteria for this type Dentinogenesis Imperfecta is incorrect. Shows bulbous crowns and constriction of tooth at the cementoenamel junction, which is not present in this case Amelogenesis Imperfecta, Hypomaturation with Taurodontism is incorrect. The clinical presentations of this case are not indicative of this type. Also, on the radiograph no signs of Taurodontism Amelogenesis Imperfecta, Hypocalcified type is incorrect. Note that in this type of Amelogenesis Imperfecta, teeth look normal at the time of eruption Regional Odontodysplasia is incorrect. This type of dental anomaly is only present in a segment of the oral cavity.

Patient: A 16-year-old male Chief complaint: I want to fix my yellow teeth. My teeth were yellow since they erupted. Current Findings: Clinically the patient's teeth show thin enamel with a yellowish-brown color. You take a radiograph, shown here Based on the clinical and radiographic evaluation of this patient what is the most likely diagnosis? A) Amelogenesis Imperfecta B) Hypomaturation with Taurodontism C) Regional Odontodysplasia D) Dentinogenesis Imperfecta E) Amelogenesis Imperfecta, Hypoplastic type F) Amelogenesis Imperfecta, Hypocalcified type

E) Temporalis muscle EXPLANATION: Temporalis muscle is correct. The normal structure noted in the image is the coronoid process. The temporalis muscle is a fan-shaped muscle on each side of the head that fills the temporal fossa with insertion on the coronoid process. Buccinator muscle is incorrect. The origin of this muscle is the alveolar process of maxilla, buccinator ridge of mandible, and pterygomandibular raphe. The insertion is the modiolus, and blends with muscles of the upper lip. Lateral pterygoid muscle is incorrect. The origin of this muscle is as follows: Superior head: infratemporal surface of sphenoid bone Inferior head: lateral surface of lateral pterygoid plate The insertion is: Superior head: articular disc and anterior aspect of the TMJ capsule Inferior head: pterygoid fovea Medial pterygoid muscle is incorrect. The origin of this muscle is as follows: Deep head: medial side of lateral pterygoid plate behind the upper teeth Superficial head: pyramidal process of palatine bone and maxillary tuberosity The insertion is the medial aspect of the angle of the mandible Masseter muscle is incorret. The origin of the muscle is zygomatic arch and maxillary process of zygomatic bone and the insertion is the angle and lateral surface of ramus of mandible.

Patient: A 16-year-old male Chief complaint: I want to fix my yellow teeth. My teeth were yellow since they erupted. Current Findings: Clinically the patient's teeth show thin enamel with a yellowish-brown color. You take a radiograph, shown here In the panoramic image, which muscle is attached to the normal structure noted with a black arrow? A) Lateral pterygoid muscle B) Buccinator muscle C) Medial pterygoid muscle D) Masseter muscle E) Temporalis muscle

A) The greater palatine nerve C) Branches of V2 D) The superior alveolar nerves EXPLANATION: Branches of V2 is correct. General sensory innervation of the mucosa in the maxillary region, including covering the hard palate and lining the maxillary sinuses, would from branches of the maxillary nerve (V2) The greater palatine nerve is correct. The greater palatine nerve runs from the greater palatine foramen anteriorly along the hard palate and provides general sensory innervation to the mucosa there The superior alveolar nerves is correct. The superior alveolar nerves pass through the maxillary sinus on their way to the tooth roots and supply the mucosa lining the sinus Branches of V3 is incorrect. V3 (the mandibular nerve) does not distribute to the maxillary region The lesser palatine nerve is incorrect. The lesser palatine nerve supplies the soft palate, not the hard palate

Patient: A 16-year-old male Chief complaint: I want to fix my yellow teeth. My teeth were yellow since they erupted. Current Findings: Clinically the patient's teeth show thin enamel with a yellowish-brown color. You take a radiograph, shown here The mucosa covering the area indicated by the red arrow would receive general sensory innervation from: (Select all that apply.) A) The greater palatine nerve B) The lesser palatine nerve C) Branches of V2 D) The superior alveolar nerves E) Branches of V3

B) Is part of the maxilla D) Is the roof of the oral cavity E) Is the hard palate EXPLANATION: The roof of the oral cavity is correct. The hard palate, indicated by the arrow, forms the roof of the oral cavity The hard palate is correct Part of the maxilla is correct. The region of the hard palate indicated by the arrow is part of the maxilla Forms the roof of the maxillary sinus is incorrect. The indicated structure forms the floor, not the roof, of the maxillary sinus Part of the sphenoid is incorrect. The sphenoid bone does not contribute to the hard palate

Patient: A 16-year-old male Chief complaint: I want to fix my yellow teeth. My teeth were yellow since they erupted. Current Findings: Clinically the patient's teeth show thin enamel with a yellowish-brown color. You take a radiograph, shown here The structure indicated by the red arrow: (Select all that apply.) A) Is part of the sphenoid. B) Is part of the maxilla. C) Forms the roof of the maxillary sinus. D) Is the roof of the oral cavity. E) Is the hard palate.

C) Pneumatization of the alveolar process by the maxillary sinus EXPLANATION: Pneumatization of the alveolar process by the maxillary sinus is correct. If you follow the floor of the maxillary sinus, you can see that the floor of the right maxillary sinus drops between the second premolar and first molar which is a result of pneumatization of the alveolar process by the maxillary sinus Odontogenic keratocyst is incorrect. The area noted is a variant of normal anatomy, not a pathology Radicular cyst is incorrect. The area noted is a variant of normal anatomy, not a pathology Antral pseudocyst is incorrect. The area noted is a variant of normal anatomy, not a pathology. Also, antral pseudocysts are well-defined radiopacity, not radiolucency

Patient: A 16-year-old male Chief complaint: I want to fix my yellow teeth. My teeth were yellow since they erupted. Current Findings: Clinically the patient's teeth show thin enamel with a yellowish-brown color. You take a radiograph, shown here What is the radiolucency area noted with a white arrow in the panoramic image? A) Odontogenic keratocyst B) Antral pseudocyst C) Pneumatization of the alveolar process by the maxillary sinus D) Radicular cyst

A) Hyoid bone B) Epiglottis EXPLANATION: Hyoid bone is correct. The hyoid bone appears as a horizontal radiopacity inferior to the mandible at each side Epiglottis is correct. The epiglottis appears as radiopacity posterior to the dorsum of tongue Ear lobe is incorrect. The ear lobe is located more superior compared to the area selected in the panoramic image Right hyoid bone is incorrect. There is only one hyoid bone which is imaged twice due to the panoramic technique Cervical vertebrae is incorrect. The cervical vertebrae are not imaged on the right side of the panoramic image

Patient: A 16-year-old male Chief complaint: I want to fix my yellow teeth. My teeth were yellow since they erupted. Current Findings: Clinically the patient's teeth show thin enamel with a yellowish-brown color. You take a radiograph, shown here Which structure(s) are located within the white oval? (Select all that apply.) A) Hyoid bone B) Epiglottis C) Cervical vertebrae D) Ear lobe E) Right hyoid bone

B) The patient is late in establishing a dental home. It should be established no later than 12 months of age

Patient: A 2 year old male patient presents to your clinic with his mother for a new patient exam According to the American Academy of Pediatric Dentistry Guidelines, which of the following statements is true with regards to this patient and establishing a dental home? A) The patient is late in establishing a dental home. It should be established no later than 6 months of age. B) The patient is late in establishing a dental home. It should be established no later than 12 months of age. C) The patient is late in establishing a dental home. It should be established no later than 18 months of age. D) The patient has established a dental home on time. It should be established no later than 24 months of age. E) The patient has established a dental home on time. It should be established no later than 30 months of age.

B) 0.25 mg

Patient: A 2 year old male patient presents to your clinic with his mother for a new patient exam Due to the patient's high caries risk status, you decide that dietary fluoride supplementation is indicated. What is the daily dose of fluoride supplementation you would prescribe to this patient according to the American Academy of Pediatric Dentistry Dietary Fluoride Supplementation Schedule? A) 0.1 mg B) 0.25 mg C) 0.5 mg D) 0.75 mg E) 1.0 mg

D: Both A and B

Patient: A 2 year old male patient presents to your clinic with his mother for a new patient exam. Chief complaint: Per his mother: "My son has never had a dental exam before and needs one. I think he has cavities." Background/or patient history: You collect the following information in your history-taking: The patient was born in the Dominican Republic, immigrated to the U.S. 6 months ago The family lives in a non-fluoridated community Mom reports that the patient brushes 2x a day on his own using a non-fluoridated toothpaste Primary language for family is Spanish; Mom does not have very high oral health literacy Patient's medical history is significant for Autism Spectrum Disorder Current Findings: The patient has cervical decalcification across teeth: C, D, E, F, G, H, M, N, O, P, Q, and R. You detect and visualize generalized plaque across the patient's dentition. Based on the collected history, which of the following factors in the patient's history would be categorized as a "high risk factor" for caries according to the American Academy of Pediatric Dentistry? A: Patient has visible plaque on his dentition B: Patient and family have low oral health literacy C: Brushing twice a day unsupervised with a non-fluoridated toothpaste D: Both A and B E: All the above

D: Both A and B

Patient: A 2 year old male patient presents to your clinic with his mother for a new patient exam. Chief complaint: Per his mother: "My son has never had a dental exam before and needs one. I think he has cavities." Background/or patient history: You collect the following information in your history-taking: The patient was born in the Dominican Republic, immigrated to the U.S. 6 months ago The family lives in a non-fluoridated community Mom reports that the patient brushes 2x a day on his own using a non-fluoridated toothpaste Primary language for family is Spanish; Mom does not have very high oral health literacy Patient's medical history is significant for Autism Spectrum Disorder Current Findings: The patient has cervical decalcification across teeth: C, D, E, F, G, H, M, N, O, P, Q, and R. You detect and visualize generalized plaque across the patient's dentition. Based on the collected history, which of the following factors in the patient's history would be categorized as a "moderate risk factor" for caries according to the American Academy of Pediatric Dentistry? A: The patient recently immigrated to the United States from the Dominican Republic B: The patient has special health care needs C: The patient has cervical decalcification across his maxillary and mandibular anterior dentition D: Both A and B E: All the above

D) The "red spot" would blanch --> Upon pressing the capillaries, the petechiae would empty of blood, blanching the tissue EXPLANATION: It would be painful for the patient. Incorrect. Petechiae are usually not painful. It would feel like a hard nodule. Incorrect. Capillaries are soft and easily compressible. It would feel like a vesicle. Incorrect. Capillaries are soft and easily compressible. It is not like a vesicle which would designate a vesicular disease.

Patient: A 53-year-old female Chief complaint: "I have red spots inside my mouth." Current Findings: The patient has similar spots on her skin. What would happen if you compressed the affected area with a finger? A) It would feel like a hard nodule. B) It would feel like a vesicle. C) It would be painful for the patient. D) The "red spot" would blanch

E) INR --> tests coagulation all other tests are either NOT involved in coagulation or do not determine the coagulation potential

Patient: A 53-year-old female Chief complaint: "I have red spots inside my mouth." Current Findings: The patient has similar spots on her skin. Which of the following lab tests you have learned about might help you confirm your initial diagnosis? A) RBC count B) WBC count C) Hematocrit D) Hemoglobin E) INR

D) Platelets EXPLANATION: Submucosal bleeding may be due to reduced number of platelets or blood and tissue components involved in the coagulation cascade --> all others are NOT involved in bleeding

Patient: A 53-year-old female Chief complaint: "I have red spots inside my mouth." Current Findings: The patient has similar spots on her skin. Which of the following structures you have learned about would be responsible for the patient's symptoms? A) Red blood cells B) Lymphocytes C) Macrophages D) Platelets E) Neutrophils

A) Capillaries --> Subcutaneous bleeding from tiny blood vessels would give such a clinical appearance EXPLANATION: Oral mucosal epithelium is incorrect. Oral epithelium does not appear red. Minor salivary glands is incorrect. Minor salivary glands are elevated and translucent, not red. Arteries is incorrect. There are no larger arteries distributed the way the lesions appear, only smaller blood vessels. Veins is incorrect. There are no large veins that are distributed in the region, only small blood vessels

Patient: A 53-year-old female Chief complaint: "I have red spots inside my mouth." Current Findings: The patient has similar spots on her skin. Which of the following tissues could be the source of the pathological feature? A) Capillaries B) Veins C) Minor salivary glands D) Arteries E) Oral mucosal epithelium

A) The patient's upper left quadrant EXPLANATION: The patient's upper left quadrant is correct. Looking at the panoramic film, you can see that a primary molar is missing from the patient's upper left quadrant. There is no space since the extraction was done three years ago and tooth#14 most likely had not erupted yet. In the time that you haven't seen the patient, tooth#14 would have erupted mesially and closed any space

Patient: A 6 year old female patient presents to your clinic with his mother for a follow up appointment post emergency treatment. Chief complaint: Per his mother: "My daughter has been doing well since the extraction appointment 3 years ago. We moved away to Europe after the extraction appointment and just moved back to the States so we were unable to come back for a follow up appointment until now." Background/or patient history: Dental History: The patient was seen at your office 3 years ago for an emergency extraction appointment of a posterior primary molar. Per your notes from 3 years ago: "The tooth in question displayed symptoms of nocturnal and spontaneous pain, pain upon percussion, and class 2 mobility." Current Findings: panoramic film taken: Based on the collected history and panoramic film, which quadrant in the oral cavity did you most likely extract the primary molar from this patient three years ago? A) The patient's upper left quadrant B) The patient's lower right quadrant C) The patient's upper right quadrant D) The patient's lower left quadrant

C) The patient's dental age is greater than her chronological age EXPLANATION: The patient's chronological age is 6. Dentally, the patient has already lost the anterior maxillary and mandibular primary teeth except for tooth#D. This exfoliation pattern would place the patient's dental age around 8 years of age

Patient: A 6 year old female patient presents to your clinic with his mother for a follow up appointment post emergency treatment. Chief complaint: Per his mother: "My daughter has been doing well since the extraction appointment 3 years ago. We moved away to Europe after the extraction appointment and just moved back to the States so we were unable to come back for a follow up appointment until now." Background/or patient history: Dental History: The patient was seen at your office 3 years ago for an emergency extraction appointment of a posterior primary molar. Per your notes from 3 years ago: "The tooth in question displayed symptoms of nocturnal and spontaneous pain, pain upon percussion, and class 2 mobility." Current Findings: panoramic film taken: Based on the panoramic radiograph and the patient's primary dentition exfoliation pattern, please choose the statement that best describes the relationship between the patient's dental age and chronological age A) The patient's dental age is equal to her chronological age B) The patient's dental age is less than her chronological age C) The patient's dental age is greater than her chronological age

C) Irreversible Pulpitis EXPLANATION: Irreversible Pulpitis is correct. The patient complained of spontaneous, nocturnal pain as well as pain upon percussion and abnormal mobility, which are all signs of irreversible pulpitis **tooth candidate for non-vital pulp therapy Reversible Pulpitis is incorrect. Reversible pulpitis would demonstrate more provoked pain that is shorter in duration Normal is incorrect. A normal pulp would not display any signs of pulpitis

Patient: A 6 year old female patient presents to your clinic with his mother for a follow up appointment post emergency treatment. Chief complaint: Per his mother: "My daughter has been doing well since the extraction appointment 3 years ago. We moved away to Europe after the extraction appointment and just moved back to the States so we were unable to come back for a follow up appointment until now." Background/or patient history: Dental History: The patient was seen at your office 3 years ago for an emergency extraction appointment of a posterior primary molar. Per your notes from 3 years ago: "The tooth in question displayed symptoms of nocturnal and spontaneous pain, pain upon percussion, and class 2 mobility." Current Findings: panoramic film taken: Based on the patient history collected regarding the extracted tooth three years ago, what is the most likely pulpal status diagnosis of the extracted tooth? A) Normal B) Reversible Pulpitis C) Irreversible Pulpitis

D) Vertical EXPLANATION: Vertical is correct. Congenital syphilis is spread by transmission from mother to child. Horizontal is incorrect. This is transmission between people of the same generation. Vector is incorrect. This is transmission when a living organism spreads the infection. Autosomal dominant is incorrect. Congenital syphilis is not an inherited genetic condition.

Patient: Female, 22 years old Chief complaint: "I am here for a dental check-up" Background/or patient history: Patient is 18 weeks pregnant. She was diagnosed with syphilis 5 days ago and is being treated with medication at this time. Current Findings: Moderate plaque and mild gingivitis If the fetus is found to also be infected, what type of transmission of disease is this? A) Vector B) Horizontal C) Autosomal dominant D) Vertical

B) Treponema pallidum EXPLANATION: Treponema pallidum is correct. Syphilis is a sexually transmitted disease caused by the bacteria Treponema pallidum, which is a spirochete. Bartonella henselae is incorrect. Syphilis is not caused by this proteobacterium. B. henselae causes cat scratch disease. Herpes Simplex is incorrect. Syphilis is not caused by a virus. Histoplasma is incorrect. Syphilis is not caused by a fungus

Patient: Female, 22 years old Chief complaint: "I am here for a dental check-up" Background/or patient history: Patient is 18 weeks pregnant. She was diagnosed with syphilis 5 days ago and is being treated with medication at this time. Current Findings: Moderate plaque and mild gingivitis The patient has an infection from which organism? A) Bartonella henselae B) Treponema pallidum C) Herpes Simplex D) Histoplasma

D) Mulberry molar EXPLANATION: Hide Feedback Mulberry molar is correct. Congenital syphilis can lead to globular enamel on the occlusal surface that resembles mulberries or raspberries. Microcephaly is incorrect. Not a finding with congenital syphilis. Hypoplasia of the zygoma is incorrect. Not a finding with congenital syphilis. Retruded mandible is incorrect. Not a finding with congenital syphilis.

Patient: Female, 22 years old Chief complaint: "I am here for a dental check-up" Background/or patient history: Patient is 18 weeks pregnant. She was diagnosed with syphilis 5 days ago and is being treated with medication at this time. Current Findings: Moderate plaque and mild gingivitis What morphologic change would commonly be seen in the infant of this patient? A) Retruded mandible B) Microcephaly C) Hypoplasia of the zygoma D) Mulberry molar

D) Chancre EXPLANATION: Chancre is correct. The initial stage of syphilis is usually a chancre. Aphthous ulcer is incorrect. Aphthous ulcers are not associated with syphillis. Blister is incorrect. Blister formation is not common with syphilis. Macule is incorrect. Change of color is not associated with the first stage of syphilis. In secondary syphilis you may get macules on the skin.

Patient: Female, 22 years old Chief complaint: "I am here for a dental check-up" Background/or patient history: Patient is 18 weeks pregnant. She was diagnosed with syphilis 5 days ago and is being treated with medication at this time. Current Findings: Moderate plaque and mild gingivitis What oral lesion would be common in the initial stage of illness in this patient? A) Blister B) Aphthous ulcer C) Macule D) Chancre

A) Use aluminum-sulfate impregnated retraction cords EXPLANATION: Use aluminum-sulfate impregnated retraction cords. Correct. Epinephrine is contraindicated in patients with acute coronary syndrome.

Patient: JD, female, 65 yrs old Chief complaint: Two teeth are worn down extensively and JD is unhappy with her looks. Background/or patient history: JD has a history of unstable angina and is currently on aspirin, nitroglycerin, metoprolol, and nifedipine. Current Findings: JD needs a procedure for dental crowns. You notice the patient has mild gingival enlargement of her papillary tissue Prior to treatment, you notice that you have epinephrine-impregnated retraction cords and aluminum-sulfate impregnated retraction cords in the clinic. What do you do next? A) Use aluminum-sulfate impregnated retraction cords B) Either epinephrine-impregnated retraction cords or aluminum-sulfate impregnated retraction cords can be used. C) Use epinephrine-impregnated retraction cords. D) Neither epinephrine-impregnated retraction cords nor aluminum-sulfate impregnated retraction cords can be used.

A) Nifedipine EXPLANATION: Nifedipine is correct. Calcium channel blockers are known to cause gingival hyperplasia.

Patient: JD, female, 65 yrs old Chief complaint: Two teeth are worn down extensively and JD is unhappy with her looks. Background/or patient history: JD has a history of unstable angina and is currently on aspirin, nitroglycerin, metoprolol, and nifedipine. Current Findings: JD needs a procedure for dental crowns. You notice the patient has mild gingival enlargement of her papillary tissue What could be causing the patient's gingival condition? A) Nifedipine B) Metoprolol C) Aspirin D) Nitroglycerin

D) Recommend the patient discuss an alternative anti-hypertensive to replace nifedipine with her PMD

Patient: JD, female, 65 yrs old Chief complaint: Two teeth are worn down extensively and JD is unhappy with her looks. Background/or patient history: JD has a history of unstable angina and is currently on aspirin, nitroglycerin, metoprolol, and nifedipine. Current Findings: JD needs a procedure for dental crowns. You notice the patient has mild gingival enlargement of her papillary tissue. Before the patient leaves, which of the following do you recommend? A) Recommend the patient discontinue using nifedipine without discussing with her PMD. B) Recommend the patient continue using nifedipine. C) Recommend the patient manage the side effects of the calcium channel blocker by switching to another pharmaceutical agent. D) Recommend the patient discuss an alternative anti-hypertensive to replace nifedipine with her PMD. E) Recommend the patient continue using nitroglycerin

B) Panoramic EXPLANATION: Panoramic is correct. Panoramic radiographs can show the maxillary and mandibular third molars entirely, as well as the relationship of the mandibular third molars with the inferior alveolar canals. Periapical radiograph of the molars is incorrect. Considering the location of the third molars, acquiring a third molar periapical radiograph is not easy. Also, a periapical radiograph may not show the relationship of the mandibular third molars with the inferior alveolar canal. Bitewings is incorrect. This radiograph does not provide enough information to show the entire third molars. Cone beam CT scan is incorrect. The main principle of radiographic prescription is ALARA (as low as reasonably achievable). Because of this principle a CBCT scan MUST not be used as the first radiographic modality and is only indicated if the result of clinical evaluation or conventional radiographs requires more information that can only be acquired by 3D imaging.

Patient: Male, 25 year-old Chief complaint: Patient "I have pain in my wisdom teeth" Background/or patient history: Patient reports pain in third molars, which is more significant on the mandibular right third molar and maxillary left third molars. There is no history of extraction of third molars. Current Findings: On clinical evaluation, maxillary third molars are erupted. There are caries on the crown of the maxillary left third molar. The mandibular left third molar is impacted. Part of the crown of the mandibular right third molar is erupted in the mouth. The tissue overlying the crown appears to be inflamed. What is the best radiographic modality for evaluation of the patient's chief complaint? A) Periapical radiograph of the molars B) Panoramic C) Bitewings D) Cone beam CT scan

A) Maxillary nerve B) Posterior superior alveolar nerve C) Inferior alveolar nerve D) Mandibular nerve E) CN V

Patient: Male, 25 year-old Chief complaint: Patient "I have pain in my wisdom teeth" Background/or patient history: Patient reports pain in third molars, which is more significant on the mandibular right third molar and maxillary left third molars. There is no history of extraction of third molars. Current Findings: On clinical evaluation, maxillary third molars are erupted. There are caries on the crown of the maxillary left third molar. The mandibular left third molar is impacted. Part of the crown of the mandibular right third molar is erupted in the mouth. The tissue overlying the crown appears to be inflamed. Which nerve(s) is/are responsible for the pain the patient is feeling? Select all that apply. A) Maxillary nerve B) Posterior superior alveolar nerve C) Inferior alveolar nerve D) Mandibular nerve E) CN V

B) Apoptosis EXPLANATION: Necrosis is incorrect. Necrosis would not clinically appear as a leukoplakia on the lip, and would more likely be an ulcer. Also, necrosis is not a regulated process by the cell, and the response to elimination of cells damaged by UV radiation is normally apoptosis. Autophagy is incorrect. While autophagy is used to eliminate cells, this is typically done under the setting of lack of resources or starvation, not as a result of sun damage. Mitosis is incorrect. Mitosis would not lead to elimination of cells but rather would lead to replication of the cell.

Patient: Male, 72 years old Chief complaint: "I am here for a dental check-up" Background/or patient history: Retired construction worker. Lives in Florida 9 months out of the year. Current Findings: You note that he has sun damage of the skin of his face and a 0.7 x 0.9 cm white, atrophic and scaly lesion of the lower lip vermilion. The lip lesion appears to be related to the patient's history of outdoor activity. What process would the body typically use to try to eliminate the cells prior to them developing into the lesion described on the lip? A) Mitosis B) Apoptosis C) Autophagy D) Necrosis

D) Actinic cheilitis EXPLANATION: Actinic cheilitis is correct. Solar damage to the lip is pre-malignant (epithelial dysplasia) and is called actinic cheilitis. Herpes labialis is incorrect. This would demonstrate either a vesicle or a ruptured vesicle with crusting or scab which is not described here. Lichen planus is incorrect. While lichen planus may appear as leukoplakia, it more often appears as reticular striae or plaque-like in appearance rather than scaly. Condyloma acuminatum is incorrect. HPV lesions like condyloma are usually raised and have a rough warty appearance clinically

Patient: Male, 72 years old Chief complaint: "I am here for a dental check-up" Background/or patient history: Retired construction worker. Lives in Florida 9 months out of the year. Current Findings: You note that he has sun damage of the skin of his face and a 0.7 x 0.9 cm white, atrophic and scaly lesion of the lower lip vermilion. What is the most likely diagnosis of the lip lesion? A) Lichen planus B) Condyloma acuminatum C) Herpes labialis D) Actinic cheilitis

E) Horizontal bone loss on the distal aspect

Patient: Mr. David Kane is 35-years-old Chief complaint: "My gums bleed when I brush and I noticed that my teeth have been shifting in the last few years." Background/or patient history: David reports that his parents lost most of their teeth and are using complete/partial dentures. He has a family history of cardiovascular disease. He has no known drug allergies but is currently taking atenolol to control his blood pressure. He reports his last "cleaning" was about 14 months ago. Smokes: 1 pack of cigarettes/day for 10 years. Current Findings: Upon the intraoral exam, you note 12% of the surfaces covered with mild plaque. Clinically, 50% of the sites are inflamed. The interdental papillae are edematous and erythematous. There are generalized probing depths ranging from 2-4 mm with increased PD of 4-8mm in the first molars and incisors regions. The radiographic bone loss is 3-8 mm (>50%) limited to the teeth # 3, 7-10, 14, 19, and 30 on the radiographs. Using a Nabers probe, you find a horizontal defect in the buccal furcation on the buccal of #30, which penetrates through the furcation but is not visible clinically. All of his teeth are present except for his 3rd molars which were removed during college. Vitals: 140/95 RAS, 100 pulse All the following radiographic features can be identified on #30 EXCEPT: A) Vertical bone loss on the distal aspect B) Interradicular bone loss on the buccal aspect C) Vertical bone loss on the mesial aspect D) Adverse crown-to-root ratio E) Horizontal bone loss on the distal aspect

D) Medical consult, oral hygiene instructions, smoking cessation counseling, SRP with antibiotic therapy, and re-evaluation

Patient: Mr. David Kane is 35-years-old Chief complaint: "My gums bleed when I brush and I noticed that my teeth have been shifting in the last few years." Background/or patient history: David reports that his parents lost most of their teeth and are using complete/partial dentures. He has a family history of cardiovascular disease. He has no known drug allergies but is currently taking atenolol to control his blood pressure. He reports his last "cleaning" was about 14 months ago. Smokes: 1 pack of cigarettes/day for 10 years. Current Findings: Upon the intraoral exam, you note 12% of the surfaces covered with mild plaque. Clinically, 50% of the sites are inflamed. The interdental papillae are edematous and erythematous. There are generalized probing depths ranging from 2-4 mm with increased PD of 4-8mm in the first molars and incisors regions. The radiographic bone loss is 3-8 mm (>50%) limited to the teeth # 3, 7-10, 14, 19, and 30 on the radiographs. Using a Nabers probe, you find a horizontal defect in the buccal furcation on the buccal of #30, which penetrates through the furcation but is not visible clinically. All of his teeth are present except for his 3rd molars which were removed during college. Vitals: 140/95 RAS, 100 pulse In PROPER SEQUENCE, what treatment plan would you recommend? A) SRP, medical consult, and antibiotic therapy B) Smoking cessation counseling,SRP, re-evaluation, evaluate occlusion and antibiotic therapy C) SRP with antibiotic therapy and re-evaluation D) Medical consult, oral hygiene instructions, smoking cessation counseling, SRP with antibiotic therapy, and re-evaluation E) Oral hygiene instructions, SRP, antibiotic therapy and medical consult

E) Molar/Incisor pattern Stage III Grade C Periodontitis

Patient: Mr. David Kane is 35-years-old Chief complaint: "My gums bleed when I brush and I noticed that my teeth have been shifting in the last few years." Background/or patient history: David reports that his parents lost most of their teeth and are using complete/partial dentures. He has a family history of cardiovascular disease. He has no known drug allergies but is currently taking atenolol to control his blood pressure. He reports his last "cleaning" was about 14 months ago. Smokes: 1 pack of cigarettes/day for 10 years. Current Findings: Upon the intraoral exam, you note 12% of the surfaces covered with mild plaque. Clinically, 50% of the sites are inflamed. The interdental papillae are edematous and erythematous. There are generalized probing depths ranging from 2-4 mm with increased PD of 4-8mm in the first molars and incisors regions. The radiographic bone loss is 3-8 mm (>50%) limited to the teeth # 3, 7-10, 14, 19, and 30 on the radiographs. Using a Nabers probe, you find a horizontal defect in the buccal furcation on the buccal of #30, which penetrates through the furcation but is not visible clinically. All of his teeth are present except for his 3rd molars which were removed during college. Vitals: 140/95 RAS, 100 pulse The MOST appropriate diagnosis for this patient is: A) Generalized Stage II Grade B Periodontitis B) Periodontitis Associated with Systemic Disease C) Molar/Incisor pattern Stage IV Grade B Periodontitis D) Generalized Stage II Grade C Periodontitis E) Molar/Incisor pattern Stage III Grade C Periodontitis

B) Grade II

Patient: Mr. Thomas Curry is a 55-year-old healthy lighthouse keeper. Chief complaint: "For the past two months, my gums have been 'itchy'. Brushing my teeth does not improve the situation." Background/or patient history: A review of systems for Thomas revealed Type II Diabetes Mellitus and a family history for cardiovascular disease. He has no known drug allergies but is currently taking Metformin to control his Type II Diabetes Mellitus. He reports his last "cleaning" was about 2 years ago. Current Findings: An extraoral exam revealed no significant findings. Upon the intraoral exam, you note 35% of the surfaces covered with mild plaque. Clinically, 40% of the sites are inflamed. There are generalized probing depths ranging from 3-7 mm and 2-6 mm clinical attachment loss. Using a Naber's probe, you find that the instrument detects a furcation defect on the distal lingual of #14 which penetrates the space but does NOT exit on the buccal aspect. His radiographs show bone loss of 40%. Approximately 60% of the sites bleed upon probing, and his plaque score is 65%. Vitals: 130/85 RAS, 80 pulse. HbA1c: 6.8%. Which of the following best describes the degree of the furcation involvement on the distal aspect of #14? A) Grade I B) Grade II C) Grade III D) Grade IV

A) Reinstitute scaling and root planing for affected sites in conjunction with local drug delivery (LDD) antibiotics

Patient: Mr. Thomas Curry is a 55-year-old healthy lighthouse keeper. Chief complaint: "For the past two months, my gums have been 'itchy'. Brushing my teeth does not improve the situation." Background/or patient history: A review of systems for Thomas revealed Type II Diabetes Mellitus and a family history for cardiovascular disease. He has no known drug allergies but is currently taking Metformin to control his Type II Diabetes Mellitus. He reports his last "cleaning" was about 2 years ago. Current Findings: An extraoral exam revealed no significant findings. Upon the intraoral exam, you note 35% of the surfaces covered with mild plaque. Clinically, 40% of the sites are inflamed. There are generalized probing depths ranging from 3-7 mm and 2-6 mm clinical attachment loss. Using a Naber's probe, you find that the instrument detects a furcation defect on the distal lingual of #14 which penetrates the space but does exit on the buccal aspect. His radiographs show bone loss of 40%. Approximately 60% of the sites bleed upon probing, and his plaque score is 65%. Vitals: 130/85 RAS, 80 pulse. HbA1c: 6.8% Following initial therapy, there are isolated residual probing depths ranging from 2-5 mm on tooth #27. The deepest pockets of 5 mm show bleeding on probing on the mesial and distal on tooth #27. The plaque score is 20%. The MOST appropriate follow-up therapy is to: A) Reinstitute scaling and root planing for affected sites in conjunction with local drug delivery (LDD) antibiotics B) Reinstitute scaling and root planing for affected sites in conjunction with OHI C) Refer this patient for periodontal surgery D) Reinforce home care and schedule a 3-month periodontal maintenance interval E) Reinstitute scaling and root planing for affected sites in conjunction with systemic anti-collagenase

C) Oral hygiene instructions, SRP, and re-evaluation

Patient: Mr. Thomas Curry is a 55-year-old healthy lighthouse keeper. Chief complaint: "For the past two months, my gums have been 'itchy'. Brushing my teeth does not improve the situation." Background/or patient history: A review of systems for Thomas revealed Type II Diabetes Mellitus and a family history for cardiovascular disease. He has no known drug allergies but is currently taking Metformin to control his Type II Diabetes Mellitus. He reports his last "cleaning" was about 2 years ago. Current Findings: An extraoral exam revealed no significant findings. Upon the intraoral exam, you note 35% of the surfaces covered with mild plaque. Clinically, 40% of the sites are inflamed. There are generalized probing depths ranging from 3-7 mm and 2-6 mm clinical attachment loss. Using a Naber's probe, you find that the instrument detects a furcation defect on the distal lingual of #14 which penetrates the space but does exit on the buccal aspect. His radiographs show bone loss of 40%. Approximately 60% of the sites bleed upon probing, and his plaque score is 65%. Vitals: 130/85 RAS, 80 pulse. HbA1c: 6.8%. In PROPER SEQUENCE, what treatment plan would you recommend? A) SRP, antibiotic therapy and medical consult B) Remove plaque and debris, medical consult, antibiotic therapy and re-evaluation C) Oral hygiene instructions, SRP, and re-evaluation D) Oral hygiene instructions, SRP, and antibiotic therapy E) SRP, evaluate occlusion, and antibiotic therapy

E) Generalized Stage III Grade B Periodontitis

Patient: Mr. Thomas Curry is a 55-year-old healthy lighthouse keeper. Chief complaint: "For the past two months, my gums have been 'itchy'. Brushing my teeth does not improve the situation." Background/or patient history: A review of systems for Thomas revealed Type II Diabetes Mellitus and a family history for cardiovascular disease. He has no known drug allergies but is currently taking Metformin to control his Type II Diabetes Mellitus. He reports his last "cleaning" was about 2 years ago. Current Findings: An extraoral exam revealed no significant findings. Upon the intraoral exam, you note 35% of the surfaces covered with mild plaque. Clinically, 40% of the sites are inflamed. There are generalized probing depths ranging from 3-7 mm and 2-6 mm clinical attachment loss. Using a Naber's probe, you find that the instrument detects a furcation defect on the distal lingual of #14 which penetrates the space but does exit on the buccal aspect. His radiographs show bone loss of 40%. Approximately 60% of the sites bleed upon probing, and his plaque score is 65%. Vitals: 130/85 RAS, 80 pulse. HbA1c: 6.8%. The MOST appropriate diagnosis for this patient is: A) Generalized Stage I Grade A Periodontitis B) Generalized Stage II Grade B Periodontitis C) Localized Stage II Grade A Periodontitis D) Localized Stage III Grade B Periodontitis E) Generalized Stage III Grade B Periodontitis

A) the alpha-subunit to bind GTP EXPLANATION: The alpha-subunit to bind GTP. Correct. When activated, the alpha subunit releases GDP and binds to a GTP. The alpha-subunit to bind GDP. Incorrect. GDP is bound to the inactive (not the active) alpha subunit. The hydrolysis of BetaGamma-subunits. Incorrect. There is no hydrolysis of the beta and gamma subunits. They remain attached to each other through the activation-inactivation cycle. The alpha-subunit to phosphorylate downstream targets. Incorrect. The activated alpha subunit (with GTP attached) activates an enzyme, which then may phosphorylate other compounds, but the alpha subunit does not itself phosphorylate anything.

The biologically active conformation of trimeric G-proteins requires: A) the alpha-subunit to bind GTP B) the hydrolysis of BetaGamma-subunits. C) the alpha-subunit to bind GDP. D) the alpha-subunit to phosphorylate downstream targets.

B) Chin tilted up EXPLANATION: Chin tilted up is correct. The occlusal plane appears to be flat in the panoramic image, which is a result of the chin being tilted up

The following image was acquired for this patient. What is the technical error in positioning of this patient causing this appearance? A) Chin tilted down B) Chin tilted up C) Patient motion D) Lead apron E) Tongue artifact

C) cells need large quantities of tRNA EXPLANATION: Cells need large quantities of tRNA. Correct. Tandem arrays are multiple copies of a gene found near each other on a region of DNA, but separated by non-transcribed spacer regions. The multiple copies can all be transcribed simultaneously, increasing the output of the mRNA. The tRNA gene is a simple gene. Incorrect. A simple gene has no introns and is therefore associated with only one mRNA sequence. Contrast this with complex genes. Complex genes can encode various mRNA sequences. Different cell types need different forms of tRNA. Incorrect. Tandem arrays have identical copies of a gene, so would not be expressed differently in different cells. Different forms of a protein could be produced by similar genes in a gene family (collagen or hemoglobin, for example). tRNA genes are found on the X chromosome. Incorrect. This is unrelated to the concept of tandem arrays.

The gene for tRNA is arranged in tandem arrays. This is because _____. A) tRNA genes are found on the X chromosome B) the tRNA gene is a simple gene C) cells need large quantities of tRNA D) different cell types need different forms of tRNA

B) below-normal osmolality EXPLANATION: Below-normal osmolality. Correct. Low osmolality indicates a low GFR. Above-normal osmolality. Incorrect. High osmolality indicates a high GFR. Above-normal hydrostatic pressure. Incorrect. Hydrostatic pressure of filtrate has no significant effect. Below-normal hydrostatic pressure. Incorrect. Hydrostatic pressure of filtrate has no significant effect. None of the other choices is correct; renin is secreted into the filtrate. Incorrect. There is a correct choice.

The juxtaglomerular apparatus will secrete renin into the afferent arteriole if it senses a(n) __________________ in the filtrate entering the distal convoluted tubule. A) above-normal osmolality B) below-normal osmolality C) above-normal hydrostatic pressure D) below-normal hydrostatic pressure E) None of the other choices is correct; renin is secreted into the filtrate.

D) filtrate away from the glomerulus. EXPLANATION: Blood to the glomerulus. Incorrect. This is done by the afferent arteriole. Blood away from the glomerulus. Incorrect. This is done by the efferent arteriole. Filtrate to the glomerulus. Incorrect. Filtrate is formed in the glomerulus, not brought to it. Two of the above are correct. Incorrect. Only one choice is correct.

The proximal convoluted tubule of a nephron carries: A) blood to the glomerulus. B) blood away from the glomerulus. C) filtrate to the glomerulus. D) filtrate away from the glomerulus. E) Two of the above are correct.

D) Submandibular gland fossa EXPLANATION: Submandibular gland fossa is correct. The radiolucent area noted in the panoramic radiograph is the result of the presence of a depression on the lingual surface of the bone in the area of submandibular gland fossa. Sublingual gland fossa is incorrect. This radiographic landmark does not exist. Mental foramen is incorrect. Mental foramen is the anterior limit of the inferior alveolar canal and appears as a small round radiolucency in the premolar area. Mental fossa is incorrect. The mental fossa is a depression on the labial aspect of the mandible extending laterally from the midline and above the mental ridge. Lingual foramen is incorrect. The lingual foramen is located in the midline of mandible in the region of the genial tubercles.

Which of the following describes the area noted with the white oval A? A) Mental fossa B) Mental foramen C) Sublingual gland fossa D) Submandibular gland fossa E) Lingual foramen

A) Dental caries

The most common chronic childhood disease is: A) Dental caries B) Asthma C) Obesity D) Diabetes

A) Postganglionic sympathetics that have synapsed at the superior cervical ganglion B) Postganglionic parasympathetics that have synapsed at the pterygopalatine ganglion EXPLANATION: Postganglionic parasympathetics that have synapsed at the pterygopalatine ganglion. Correct. Some of the preganglionic parasympathetics originating with the facial nerve (those found in the greater petrosal branch of the facial nerve) synapse at the pterygopalatine ganglion and then travel with branches of the maxillary nerve. Postganglionic sympathetics that have synapsed at the superior cervical ganglion. Correct. Some postganglionic sympathetics that have synapsed at the superior cervical ganglion (those found in the deep petrosal nerve) will then travel with branches of the maxillary nerve. Postganglionic parasympathetics that have synapsed at the geniculate ganglion. Incorrect. The geniculate ganglion is the sensory ganglion of the facial nerve, and parasympathetics do not synapse there. Postganglionic parasympathetics that have synapsed at the submandibular ganglion. Incorrect. Some of the parasympathetics originating with the facial nerve do synapse at the submandibular ganglion, but these then travel with branches of the mandibular nerve postganglionically, not the maxillary nerve. Preganglionic parasympathetics that have synapsed at the pterygopalatine ganglion. Incorrect. Parasympathetics that have synapsed in a ganglion are postganglionic, not preganglionic.

The nerves that provide sensory innervation to the maxillary molars have the following fibers traveling with them: Select all that apply. A) Postganglionic sympathetics that have synapsed at the superior cervical ganglion B) Postganglionic parasympathetics that have synapsed at the pterygopalatine ganglion C) Preganglionic parasympathetics that have synapsed at the pterygopalatine ganglion D) Postganglionic parasympathetics that have synapsed at the geniculate ganglion E) Postganglionic parasympathetics that have synapsed at the submandibular ganglion

C) The thickness of the alveolar wall EXPLANATION: The rate of diffusion is directly proportional to the surface area, solubility of the gas and differences in pressure but inversely proportional to the thickness of the alveolar wall

The rate of diffusion across the alveolar wall is inversely proportional to which of the following? A) The surface area for gaseous exchange B) The difference in the partial pressures of the gas C) The thickness of the alveolar wall D) The solubility of the gas

D) Ensure the composite is contacting no more than two adjacent walls

To minimize polymerization stress and shrinkage when placing a composite resin restoration, you should A) Use a composite resin with a low filler volume B) Ensure the composite is contacting opposing walls C) Place composite in large increments D) Ensure the composite is contacting no more than two adjacent walls

F) Leukocytes EXPLANATION: Leukocytes is correct. Leukocytes are white blood cells. They circulate throughout the blood stream, but ultimately perform their immunological functions in lymphoid or connective tissue. Erythrocytes is incorrect. Erythrocytes are red blood cells. They usually stay in the blood stream. Megakaryocytes is incorrect. Megakaryocytes are large cells that produce platelets. They are located in the bone marrow. Osteocytes is incorrect. Osteocytes are bone cells. Fibrocytes is incorrect. Fibrocytes are connective tissue cells. Chondrocytes is incorrect. Chondrocytes are cartilage cells.

Which of the following cell types will circulate throughout the body in the blood, then leave the circulation to perform its assigned function? A) Erythrocytes B) Megakaryocytes C) Osteocytes D) Chondrocytes E) Fibrocytes F) Leukocytes

B) It is the most cranial structure.

What is the location of the heart primordium in the 3-week old embryo? A) It is caudal to the buccopharyngeal membrane. B) It is the most cranial structure. C) There is no heart primordium in the 3-week old embryo. D) It is caudal to the cloacal membrane.

A) Gliding; D) Hinge Hinge is correct. The lower compartment of the temporomandibular joint is a hinge joint. Gliding is correct. The upper compartment of the temporomandibular joint is a gliding joint. Ball and socket is incorrect. The temporomandibular joint is not a ball and socket joint. An example of of a ball and socket joint would be the head of the humerus in glenoid fossa of the scapula. Pivot is incorrect. The temporomandibular joint is not a pivot joint. An example of a pivot joint would be the head of the radius on the capitulum of the humerus

What type of joint is encircled in red? Select all that apply A) Gliding B) Pivot C) Ball and socket D) Hinge

A) Submental nodes, B) L submandibular nodes, and D) R submandibular nodes EXPLANATION: Right submandibular nodes is correct. Submandibular lymph nodes collect lymph from the tongue and floor of the mouth. Left submandibular nodes is correct. The tumor is on the right side. It is, however, close to midline. Lymph from mucosa close to the midline drains in nodes on the right and left sides. Submental nodes is correct. Submental nodes drain lymph primarily from the tip of the tongue. This carcinoma extends towards the tip of the tongue and submental lymph nodes might contain cancer cells. Parotid nodes is incorrect. Parotid nodes drain superficial areas on the face and temporal region.

Which of the following lymph nodes are likely to contain cancer cells? (Select all correct choices) A) Submental nodes B) Left submandibular nodes C) Parotid nodes D) Right submandibular nodes

A) An occlusal class I cavity has a high C-factor, and is very susceptible to de-bonding

Which of these statements about composite resin restorations, is true? A) An occlusal class I cavity has a high C-factor, and is very susceptible to de-bonding B) A class IV restoration has the greatest potential for polymerization stress C) A buccal class V cavity has a low C-factor, and is not susceptible to de-bonding D) A class II MO cavity has a lower risk of de-bonding than a class II MOD cavity

D) Squamous cell carcinoma EXPLANATION: Squamous cell carcinoma is correct. The question describes actinic change to the lip vermillion. Patients with actinic cheilitis have increased risk of transformation to malignancy (squamous cell carcinoma). Mucous membrane pemphigoid is incorrect. MMP is an autoimmune vesiculobullous disorder which would show vesicles and ulceration not seen in this case. Sun exposure does not alter your chance of developing MMP. HPV infection is incorrect. Sun exposure does not increase your chances of developing HPV lesions and the clinical appearance would be of a raised and likely wart like appearance which is not described in this case. Herpes labialis is incorrect. The patient has no history of HSV infection. While sun exposure may trigger a herpes outbreak in a person with a history of such lesions, that is not the case seen here.

atient: Male, 72 years old Chief complaint: "I am here for a dental check-up" Background/or patient history: Retired construction worker. Lives in Florida 9 months out of the year. Current Findings: You note that he has sun damage of the skin of his face and a 0.7 x 0.9 cm white, atrophic and scaly lesion of the lower lip vermilion. What is the patient at increased risk of developing based on the clinical findings and history? A) Mucous membrane pemphigoid B) Herpes labialis C) HPV infection D) Squamous cell carcinoma


संबंधित स्टडी सेट्स

Bus 1270 Principles of Computer Information Systems C01 and C02 Test 1

View Set

Farmacología - 3er Parcial - Examenes Viejos

View Set

Unit 02 Multiple Regression Analysis

View Set

Chapter 20: Nursing Management of the Pregnancy at Risk - Selected Health Conditions and Vulnerable Populations

View Set

Lab exam 2 questions from lab manual

View Set